GRE 9367 Problem #38 (Summations + Limits)

In summary, the limit as n approaches infinity of the given expression is equal to the limit of the sum of two distinct sums, where the second sum is -(n+1)n/2. By recognizing the sum of i^2 as a Riemann sum for the function 27x^2-9x integrated between 0 and 1, the limit can be easily evaluated, resulting in the answer D) 9/2. The interval [0,1] is split into n equal intervals, with the value of x at the end of an interval being i/n. The width of each rectangle is 1/n and the height is given by the function 27x^2-9x. This allows for the
  • #1
moo5003
207
0

Homework Statement



The limit as n approaches infinity of:
The sum from i = 1 to n of:

(3/n)^3 * i^2 - (3/n)^2 * i

A) -1/6
B) 0
C) 3
D) 9/2
E) 31/6

Answer - D

The Attempt at a Solution



So, my first idea was to split the sum into two distinct sums and then take out powers of 3/n

ie:
Limit of:

27/n^3 Sum of i^2

+

Limit of:

9/n^2 Sum of -i

Now the second sum is just -(n+1)n/2 thus the second term goes to -9/2 as n goes to infinity.

This is were I'm stuck any help would be appreciated. Or suggestions on approaching this problem differently.
 
Last edited:
Physics news on Phys.org
  • #2
Why do you only have sum(i^2) in the first one. Isn't it sum(i^3)? Are you sure you've stated the problem correctly? As stated, the limit diverges.
 
  • #3
Sorry you are correct, I didnt write it out correctly. I edited the first post so it should be correct now.
 
  • #4
The sum of i^2 from 1 to n is n(n+1)(2n+1)/6. That's one way to do it. The easy way is to recognize it as a Riemann sum for the function 27x^2-9x integrated between 0 and 1.
 
  • #5
Thanks for the sum on i^2 seems to work out. I'm a little unclear how you recognize that the sum is infact a riemann sum integrated between 0 and 1.

I can see where the function is namely 27i^2-9i however could you explain in further detail how the interval is seen?
 
  • #6
Split the interval [0,1] into n equal intervals. The value of x at the end of an interval is i/n. The width of a rectangle is 1/n. The height of the rectangle corresponding to f(x)=27x^2-9x is 27(i/n)^2-9(i/n). Multiply height time width and that the contribution to the area sum from each rectangle. That's your sum.
 

What is GRE 9367 Problem #38 about?

GRE 9367 Problem #38 involves solving a summation series with limits and finding the value of the series at a specific point.

What is a summation series?

A summation series is a mathematical expression that represents the sum of a sequence of numbers. It is denoted by the Greek letter sigma (Σ) and has an upper and lower limit.

What are limits in mathematics?

Limits are a fundamental concept in calculus that describe the behavior of a function as the input approaches a certain value. It is used to analyze the behavior of functions at points where they are undefined or discontinuous.

How do you solve a summation series with limits?

To solve a summation series with limits, you first need to determine the general term of the series. Then, substitute the upper and lower limits into the general term and simplify the expression. Finally, evaluate the expression to find the sum of the series at the given limits.

Why is GRE 9367 Problem #38 important?

Solving summation series with limits is an essential skill in mathematics, particularly in calculus and advanced mathematical fields. This problem also tests your understanding of limits and series, which are crucial concepts in many areas of science and engineering.

Similar threads

  • Calculus and Beyond Homework Help
Replies
1
Views
352
  • Calculus and Beyond Homework Help
Replies
4
Views
286
  • Calculus and Beyond Homework Help
Replies
1
Views
232
  • Calculus and Beyond Homework Help
Replies
8
Views
1K
  • Calculus and Beyond Homework Help
Replies
6
Views
219
  • Calculus and Beyond Homework Help
Replies
12
Views
2K
  • Calculus and Beyond Homework Help
Replies
14
Views
513
  • Calculus and Beyond Homework Help
Replies
3
Views
988
  • Calculus and Beyond Homework Help
Replies
3
Views
541
  • Calculus and Beyond Homework Help
Replies
24
Views
1K
Back
Top